why is the limit of this expression equal to 1? The 2019 Stack Overflow Developer Survey...

Can a flute soloist sit?

Can withdrawing asylum be illegal?

Keeping a retro style to sci-fi spaceships?

Is every episode of "Where are my Pants?" identical?

Are there continuous functions who are the same in an interval but differ in at least one other point?

One-dimensional Japanese puzzle

"... to apply for a visa" or "... and applied for a visa"?

Windows 10: How to Lock (not sleep) laptop on lid close?

1960s short story making fun of James Bond-style spy fiction

First use of “packing” as in carrying a gun

Do I have Disadvantage attacking with an off-hand weapon?

Accepted by European university, rejected by all American ones I applied to? Possible reasons?

Can we generate random numbers using irrational numbers like π and e?

How to type a long/em dash `—`

How can a C program poll for user input while simultaneously performing other actions in a Linux environment?

Visa regaring travelling European country

Match Roman Numerals

Make it rain characters

Sort list of array linked objects by keys and values

Variable with quotation marks "$()"

Is it ethical to upload a automatically generated paper to a non peer-reviewed site as part of a larger research?

Huge performance difference of the command find with and without using %M option to show permissions

For what reasons would an animal species NOT cross a *horizontal* land bridge?

Why not take a picture of a closer black hole?



why is the limit of this expression equal to 1?



The 2019 Stack Overflow Developer Survey Results Are In
Announcing the arrival of Valued Associate #679: Cesar Manara
Planned maintenance scheduled April 17/18, 2019 at 00:00UTC (8:00pm US/Eastern)Finding the limit of the following expressionReforming series expression for limit of e$lim_{x rightarrow infty}left(frac{pi}{2}-tan^{-1}xright)^{Largefrac{1}{x}}$ Why aren't these two limits equal when they should be?What is the value of this limit?limit of an expressionUsing a definite integral find the value of $lim_{nrightarrow infty }(frac{1}{n}+frac{1}{n+1}+…+frac{1}{2n})$Why is the following limit operation valid?Is this expression on limit valid and/or meaningful?Why does this limit equal 0?A Problem on the Limit of an Integral












1












$begingroup$


I found something which I find confusing.



$$
lim_{nrightarrow infty} frac{n!}{n^{k}(n-k)! } =1
$$



It was something I encountered while learning probability on this webpage.










share|cite|improve this question











$endgroup$

















    1












    $begingroup$


    I found something which I find confusing.



    $$
    lim_{nrightarrow infty} frac{n!}{n^{k}(n-k)! } =1
    $$



    It was something I encountered while learning probability on this webpage.










    share|cite|improve this question











    $endgroup$















      1












      1








      1


      2



      $begingroup$


      I found something which I find confusing.



      $$
      lim_{nrightarrow infty} frac{n!}{n^{k}(n-k)! } =1
      $$



      It was something I encountered while learning probability on this webpage.










      share|cite|improve this question











      $endgroup$




      I found something which I find confusing.



      $$
      lim_{nrightarrow infty} frac{n!}{n^{k}(n-k)! } =1
      $$



      It was something I encountered while learning probability on this webpage.







      limits






      share|cite|improve this question















      share|cite|improve this question













      share|cite|improve this question




      share|cite|improve this question








      edited 2 hours ago







      billyandr

















      asked 2 hours ago









      billyandrbillyandr

      155




      155






















          2 Answers
          2






          active

          oldest

          votes


















          5












          $begingroup$

          It is rather obvious if you cancel the factorials:



          $$frac{n!}{n^{k}(n-k)! } =frac{overbrace{n(n-1)cdots (n-k+1)}^{k; factors}}{n^k}= 1cdot left(1-frac{1}{n}right)cdots left(1-frac{k-1}{n}right)stackrel{n to infty}{longrightarrow} 1$$






          share|cite|improve this answer









          $endgroup$













          • $begingroup$
            Thank you so much. I didn't know it was right there under my eyes.
            $endgroup$
            – billyandr
            1 hour ago










          • $begingroup$
            You are welcome. This "not seeing the obvious" just happens once in a while, I think, to all who do maths. So, it is good to have a math platform like this one. :-)
            $endgroup$
            – trancelocation
            1 hour ago





















          2












          $begingroup$

          $$a_n=frac{n!}{n^{k}(n-k)! }implies log(a_n)=log(n!)-k log(n)-log((n-k)!)$$



          Use Stirling approximation and continue with Taylor series to get
          $$log(a_n)=frac{k(1-k)}{2 n}+Oleft(frac{1}{n^2}right)$$ Continue with Taylor
          $$a_n=e^{log(a_n)}=1+frac{k(1-k)}{2 n}+Oleft(frac{1}{n^2}right)$$






          share|cite|improve this answer









          $endgroup$













          • $begingroup$
            This has already a slight touch of overkill, hasn't it? :-)
            $endgroup$
            – trancelocation
            1 hour ago










          • $begingroup$
            @trancelocation. You are totally right for the limit. One of my manias is to always look at the approach to the limit. Have a look at matheducators.stackexchange.com/questions/8339/… . Cheers :-)
            $endgroup$
            – Claude Leibovici
            1 hour ago














          Your Answer








          StackExchange.ready(function() {
          var channelOptions = {
          tags: "".split(" "),
          id: "69"
          };
          initTagRenderer("".split(" "), "".split(" "), channelOptions);

          StackExchange.using("externalEditor", function() {
          // Have to fire editor after snippets, if snippets enabled
          if (StackExchange.settings.snippets.snippetsEnabled) {
          StackExchange.using("snippets", function() {
          createEditor();
          });
          }
          else {
          createEditor();
          }
          });

          function createEditor() {
          StackExchange.prepareEditor({
          heartbeatType: 'answer',
          autoActivateHeartbeat: false,
          convertImagesToLinks: true,
          noModals: true,
          showLowRepImageUploadWarning: true,
          reputationToPostImages: 10,
          bindNavPrevention: true,
          postfix: "",
          imageUploader: {
          brandingHtml: "Powered by u003ca class="icon-imgur-white" href="https://imgur.com/"u003eu003c/au003e",
          contentPolicyHtml: "User contributions licensed under u003ca href="https://creativecommons.org/licenses/by-sa/3.0/"u003ecc by-sa 3.0 with attribution requiredu003c/au003e u003ca href="https://stackoverflow.com/legal/content-policy"u003e(content policy)u003c/au003e",
          allowUrls: true
          },
          noCode: true, onDemand: true,
          discardSelector: ".discard-answer"
          ,immediatelyShowMarkdownHelp:true
          });


          }
          });














          draft saved

          draft discarded


















          StackExchange.ready(
          function () {
          StackExchange.openid.initPostLogin('.new-post-login', 'https%3a%2f%2fmath.stackexchange.com%2fquestions%2f3185830%2fwhy-is-the-limit-of-this-expression-equal-to-1%23new-answer', 'question_page');
          }
          );

          Post as a guest















          Required, but never shown

























          2 Answers
          2






          active

          oldest

          votes








          2 Answers
          2






          active

          oldest

          votes









          active

          oldest

          votes






          active

          oldest

          votes









          5












          $begingroup$

          It is rather obvious if you cancel the factorials:



          $$frac{n!}{n^{k}(n-k)! } =frac{overbrace{n(n-1)cdots (n-k+1)}^{k; factors}}{n^k}= 1cdot left(1-frac{1}{n}right)cdots left(1-frac{k-1}{n}right)stackrel{n to infty}{longrightarrow} 1$$






          share|cite|improve this answer









          $endgroup$













          • $begingroup$
            Thank you so much. I didn't know it was right there under my eyes.
            $endgroup$
            – billyandr
            1 hour ago










          • $begingroup$
            You are welcome. This "not seeing the obvious" just happens once in a while, I think, to all who do maths. So, it is good to have a math platform like this one. :-)
            $endgroup$
            – trancelocation
            1 hour ago


















          5












          $begingroup$

          It is rather obvious if you cancel the factorials:



          $$frac{n!}{n^{k}(n-k)! } =frac{overbrace{n(n-1)cdots (n-k+1)}^{k; factors}}{n^k}= 1cdot left(1-frac{1}{n}right)cdots left(1-frac{k-1}{n}right)stackrel{n to infty}{longrightarrow} 1$$






          share|cite|improve this answer









          $endgroup$













          • $begingroup$
            Thank you so much. I didn't know it was right there under my eyes.
            $endgroup$
            – billyandr
            1 hour ago










          • $begingroup$
            You are welcome. This "not seeing the obvious" just happens once in a while, I think, to all who do maths. So, it is good to have a math platform like this one. :-)
            $endgroup$
            – trancelocation
            1 hour ago
















          5












          5








          5





          $begingroup$

          It is rather obvious if you cancel the factorials:



          $$frac{n!}{n^{k}(n-k)! } =frac{overbrace{n(n-1)cdots (n-k+1)}^{k; factors}}{n^k}= 1cdot left(1-frac{1}{n}right)cdots left(1-frac{k-1}{n}right)stackrel{n to infty}{longrightarrow} 1$$






          share|cite|improve this answer









          $endgroup$



          It is rather obvious if you cancel the factorials:



          $$frac{n!}{n^{k}(n-k)! } =frac{overbrace{n(n-1)cdots (n-k+1)}^{k; factors}}{n^k}= 1cdot left(1-frac{1}{n}right)cdots left(1-frac{k-1}{n}right)stackrel{n to infty}{longrightarrow} 1$$







          share|cite|improve this answer












          share|cite|improve this answer



          share|cite|improve this answer










          answered 1 hour ago









          trancelocationtrancelocation

          14.1k1829




          14.1k1829












          • $begingroup$
            Thank you so much. I didn't know it was right there under my eyes.
            $endgroup$
            – billyandr
            1 hour ago










          • $begingroup$
            You are welcome. This "not seeing the obvious" just happens once in a while, I think, to all who do maths. So, it is good to have a math platform like this one. :-)
            $endgroup$
            – trancelocation
            1 hour ago




















          • $begingroup$
            Thank you so much. I didn't know it was right there under my eyes.
            $endgroup$
            – billyandr
            1 hour ago










          • $begingroup$
            You are welcome. This "not seeing the obvious" just happens once in a while, I think, to all who do maths. So, it is good to have a math platform like this one. :-)
            $endgroup$
            – trancelocation
            1 hour ago


















          $begingroup$
          Thank you so much. I didn't know it was right there under my eyes.
          $endgroup$
          – billyandr
          1 hour ago




          $begingroup$
          Thank you so much. I didn't know it was right there under my eyes.
          $endgroup$
          – billyandr
          1 hour ago












          $begingroup$
          You are welcome. This "not seeing the obvious" just happens once in a while, I think, to all who do maths. So, it is good to have a math platform like this one. :-)
          $endgroup$
          – trancelocation
          1 hour ago






          $begingroup$
          You are welcome. This "not seeing the obvious" just happens once in a while, I think, to all who do maths. So, it is good to have a math platform like this one. :-)
          $endgroup$
          – trancelocation
          1 hour ago













          2












          $begingroup$

          $$a_n=frac{n!}{n^{k}(n-k)! }implies log(a_n)=log(n!)-k log(n)-log((n-k)!)$$



          Use Stirling approximation and continue with Taylor series to get
          $$log(a_n)=frac{k(1-k)}{2 n}+Oleft(frac{1}{n^2}right)$$ Continue with Taylor
          $$a_n=e^{log(a_n)}=1+frac{k(1-k)}{2 n}+Oleft(frac{1}{n^2}right)$$






          share|cite|improve this answer









          $endgroup$













          • $begingroup$
            This has already a slight touch of overkill, hasn't it? :-)
            $endgroup$
            – trancelocation
            1 hour ago










          • $begingroup$
            @trancelocation. You are totally right for the limit. One of my manias is to always look at the approach to the limit. Have a look at matheducators.stackexchange.com/questions/8339/… . Cheers :-)
            $endgroup$
            – Claude Leibovici
            1 hour ago


















          2












          $begingroup$

          $$a_n=frac{n!}{n^{k}(n-k)! }implies log(a_n)=log(n!)-k log(n)-log((n-k)!)$$



          Use Stirling approximation and continue with Taylor series to get
          $$log(a_n)=frac{k(1-k)}{2 n}+Oleft(frac{1}{n^2}right)$$ Continue with Taylor
          $$a_n=e^{log(a_n)}=1+frac{k(1-k)}{2 n}+Oleft(frac{1}{n^2}right)$$






          share|cite|improve this answer









          $endgroup$













          • $begingroup$
            This has already a slight touch of overkill, hasn't it? :-)
            $endgroup$
            – trancelocation
            1 hour ago










          • $begingroup$
            @trancelocation. You are totally right for the limit. One of my manias is to always look at the approach to the limit. Have a look at matheducators.stackexchange.com/questions/8339/… . Cheers :-)
            $endgroup$
            – Claude Leibovici
            1 hour ago
















          2












          2








          2





          $begingroup$

          $$a_n=frac{n!}{n^{k}(n-k)! }implies log(a_n)=log(n!)-k log(n)-log((n-k)!)$$



          Use Stirling approximation and continue with Taylor series to get
          $$log(a_n)=frac{k(1-k)}{2 n}+Oleft(frac{1}{n^2}right)$$ Continue with Taylor
          $$a_n=e^{log(a_n)}=1+frac{k(1-k)}{2 n}+Oleft(frac{1}{n^2}right)$$






          share|cite|improve this answer









          $endgroup$



          $$a_n=frac{n!}{n^{k}(n-k)! }implies log(a_n)=log(n!)-k log(n)-log((n-k)!)$$



          Use Stirling approximation and continue with Taylor series to get
          $$log(a_n)=frac{k(1-k)}{2 n}+Oleft(frac{1}{n^2}right)$$ Continue with Taylor
          $$a_n=e^{log(a_n)}=1+frac{k(1-k)}{2 n}+Oleft(frac{1}{n^2}right)$$







          share|cite|improve this answer












          share|cite|improve this answer



          share|cite|improve this answer










          answered 1 hour ago









          Claude LeiboviciClaude Leibovici

          125k1158135




          125k1158135












          • $begingroup$
            This has already a slight touch of overkill, hasn't it? :-)
            $endgroup$
            – trancelocation
            1 hour ago










          • $begingroup$
            @trancelocation. You are totally right for the limit. One of my manias is to always look at the approach to the limit. Have a look at matheducators.stackexchange.com/questions/8339/… . Cheers :-)
            $endgroup$
            – Claude Leibovici
            1 hour ago




















          • $begingroup$
            This has already a slight touch of overkill, hasn't it? :-)
            $endgroup$
            – trancelocation
            1 hour ago










          • $begingroup$
            @trancelocation. You are totally right for the limit. One of my manias is to always look at the approach to the limit. Have a look at matheducators.stackexchange.com/questions/8339/… . Cheers :-)
            $endgroup$
            – Claude Leibovici
            1 hour ago


















          $begingroup$
          This has already a slight touch of overkill, hasn't it? :-)
          $endgroup$
          – trancelocation
          1 hour ago




          $begingroup$
          This has already a slight touch of overkill, hasn't it? :-)
          $endgroup$
          – trancelocation
          1 hour ago












          $begingroup$
          @trancelocation. You are totally right for the limit. One of my manias is to always look at the approach to the limit. Have a look at matheducators.stackexchange.com/questions/8339/… . Cheers :-)
          $endgroup$
          – Claude Leibovici
          1 hour ago






          $begingroup$
          @trancelocation. You are totally right for the limit. One of my manias is to always look at the approach to the limit. Have a look at matheducators.stackexchange.com/questions/8339/… . Cheers :-)
          $endgroup$
          – Claude Leibovici
          1 hour ago




















          draft saved

          draft discarded




















































          Thanks for contributing an answer to Mathematics Stack Exchange!


          • Please be sure to answer the question. Provide details and share your research!

          But avoid



          • Asking for help, clarification, or responding to other answers.

          • Making statements based on opinion; back them up with references or personal experience.


          Use MathJax to format equations. MathJax reference.


          To learn more, see our tips on writing great answers.




          draft saved


          draft discarded














          StackExchange.ready(
          function () {
          StackExchange.openid.initPostLogin('.new-post-login', 'https%3a%2f%2fmath.stackexchange.com%2fquestions%2f3185830%2fwhy-is-the-limit-of-this-expression-equal-to-1%23new-answer', 'question_page');
          }
          );

          Post as a guest















          Required, but never shown





















































          Required, but never shown














          Required, but never shown












          Required, but never shown







          Required, but never shown

































          Required, but never shown














          Required, but never shown












          Required, but never shown







          Required, but never shown







          Popular posts from this blog

          “%fieldName is a required field.”, in Magento2 REST API Call for GET Method Type The Next...

          How to change City field to a dropdown in Checkout step Magento 2Magento 2 : How to change UI field(s)...

          夢乃愛華...